LSAT and Law School Admissions Forum

Get expert LSAT preparation and law school admissions advice from PowerScore Test Preparation.

Homework or Lesson help relating to our Accelerated or Live Online Accelerated Courses.
 acastillo
  • Posts: 4
  • Joined: May 11, 2015
|
#18595
Hi there,
I have a question page 71 game# 4 question #9
The answer is B but I chose C since my answer was a direct rule violation. Could I get an explanation?
Thanks and be easy
Art
 Morgan O'Donnell
PowerScore Staff
  • PowerScore Staff
  • Posts: 67
  • Joined: Jun 25, 2012
|
#18599
Hi Art,

Thanks for your question, and welcome to the Forum!! Can you tell us what book you are working out of? Once we get that information, we will be able to get you an explanation!

Thanks,

Morgan O'Donnell
PowerScore Test Prep
 acastillo
  • Posts: 4
  • Joined: May 11, 2015
|
#18600
Hello Morgan,
Yes, its in the Powerscore Test Preparation
The Powerscore LSAT Course
I took the weekend on-line course.
Thanks and be easy
 Andrew Ash
PowerScore Staff
  • PowerScore Staff
  • Posts: 32
  • Joined: Sep 15, 2014
|
#18605
Hi Art,

Thanks for your post!

This is a challenging advanced linear game (for those of you following along, we're looking at the four medical training sessions, which was on the June 1997 LSAT). The main thing that makes it difficult is that it's Unbalanced: we have four training sessions, and only three nurses and three doctors to teach them. This, combined with the first rule that "Each professional teaches at least once," means that one nurse is going to teach twice and the others will each teach once. In the language of Numerical Distributions, which we discuss in detail on page 126, this would be a 2-1-1 Unfixed distribution.

So that means that either Leopold or Fine could teach two sessions (Johnson can only teach one because of the fourth rule). That's why question 9 answer choice (C) actually isn't a direct rule violation and doesn't have to be false: it's possible that Leopold teaches both the first session and the third session.

The reason why answer choice (B) does have to be false is that the fourth rule, "Johnson teaches session S only," guarantees that S is paired with J - there's only one session S, so for any day that has session S, I can guarantee that J is going to be teaching it. Since the second rule guarantees that L is going to be teaching on day 3, it therefore can't be session S.

I hope this helps!

Thanks,
Andrew
 acastillo
  • Posts: 4
  • Joined: May 11, 2015
|
#18613
Thanks Andrew,
I got so caught up on solving the question that I overlooked the the question stem and the word false. Thanks for the help. At least I got the process right but no points for that. Be easy.
Art
 acastillo
  • Posts: 4
  • Joined: May 11, 2015
|
#18615
Okay sorry about that but I simply got that one wrong after further review-its early here. Okay back to the drawing board but I do understand why I got the answer wrong. Thanks.
 Morgan O'Donnell
PowerScore Staff
  • PowerScore Staff
  • Posts: 67
  • Joined: Jun 25, 2012
|
#18620
Glad we could help, Art!

Let us know if you have any further questions!!

Best,
Morgan O'Donnell
PowerScore Test Prep

Get the most out of your LSAT Prep Plus subscription.

Analyze and track your performance with our Testing and Analytics Package.